Hilberts sechstes Problem (aktuelle Antworten vernachlässigen die Tatsache, dass CU⊆UCU⊆UC_{U} \subseteq U ) [duplizieren]

(Gegenwärtige Antworten vernachlässigen die Tatsache, dass die Menge aller Konzepte ( C U ) ist eine Teilmenge von U, da alle physisch codiert sind (symbolisch dargestellt durch die physischen Ereignisse selbst (Gehirne, Computer usw.)) (Anmerkung ist das sich gegenseitig ausschließende oder, A S bedeutet, dass S als Satz des Axiomensystems A) abgeleitet (bewiesen) werden kann. Gegeben U, ( u u U ) (Nehmen Sie auch an, dass U das physikalische Universum / Multiversum usw. ist [was es trivialerweise ist, da alle Konzepte physikalische Phänomene sind {Neurotransmitter, Computer usw.}] Dies geht offensichtlich davon aus, dass der Platonismus falsch ist oder immer noch als physikalisch klassifiziert werden kann , dh ein anderes Universum im Multiversum (trivialerweise)) Auch gegeben A U st A ist ein Satz von Anfangsbedingungen und einigen Gleichungen, die den Ablauf physikalischer Ereignisse beschreiben. Diese Gleichungen sind die Einschränkungen für die Zeichenfolgen der formalen Sprache, die mit dem Axiomensystem verbunden sind (dies existiert trivialerweise, da die Reihenfolge / der Index über den Axiomen gegeben ist, st.t Offensichtlich kann man einen Code/eine Gleichung schreiben, der bzw. die die Zeichenkette von Ax.n in Ax.n+1 umwandelt, dann kann man den Code/die Gleichung induktiv verwenden, um zu jedem Ax zu gelangen, wenn den Anfangsbedingungen der Index zugeordnet ist .m. oder man kann seine Äquivalenz zur Existenz von Ursache und Wirkung beweisen. Dann ( ( A S ) ( S S U ) ) ( ( ¬ ( A S ) ) ( ¬ ( S ) ) ) , das heißt, es gibt keine einzige Aussage, die nicht aus A abgeleitet werden kann, dh es ist eine Axiomatisierung von allem (das physikalische Universum/Antwort auf Hilberts sechstes Problem). Nach Gödels Unvollständigkeitssatz ( da er offensichtlich grundlegende arithmetische Wahrheiten beweisen kann ) ( ( ( ϕ ( A ϕ A ¬ ϕ ) ) ( ϕ ( ϕ ¬ ϕ ) ) ( ϕ ( ( ϕ U ) ( ϕ U ) ) ) ) ( ( ϕ ( ϕ U ) ) ( ϕ ( ( ϕ U ) ( ϕ U ) ) ) ) ) .

Das heißt, die Menge ist sowohl unvollständig als auch inkonsistent, dies ist ein Widerspruch, da U per Definition sowohl vollständig als auch konsistent ist, so trivial, dass entweder unsere Ideen des mathematischen Formalismus keine bijektive Beziehung zu vollständigen Modellen der physikalischen Realität haben und es eine negative Lösung für Hilberts sechste gibt Problem oder Existenz und Nichtexistenz sind äquivalent. Dies würde trivialerweise auch die Existenz einer "Theorie von allem" und einer einzigen vereinheitlichenden Gleichung widerlegen, da man die Gleichung als Beschränkungen für die Zeichenfolgen in der Sprache des Axiomensystems verwenden könnte, von dem die Gleichung das einzige Axiom ist, und ( algebraisch/differentiell et al.) Manipulationen der Gleichung sind Theoreme.

Gibt es eine Möglichkeit, dieses Paradoxon zu lösen?

Dieses Papier stellt das Paradox vollständig dar. https://www.academia.edu/11102734/On_undecidable_physical_statements_in_current_mathematical_formalism (und veranschaulicht auch seine Bedeutung und seine derzeitige Unentscheidbarkeit und zeigt, dass es sich von zuvor beantworteten Fragen unterscheidet.)

Erstens könnte dies eine bessere Formatierung verwenden. Zweitens klingt Ihre Frage so: "Es gibt einen Satz, der besagt, dass jedes X, das Y ist, ein Problem Z hat. Angenommen, wir haben ein X, das Y ist. Ist es wahr, dass X das Problem Z hat?"
Sie behaupten also, es sei eine triviale Tautologie?
Ja, ich glaube, es ist eine Tautologie zu sagen, dass ein Axiomensystem, das Hilberts Problem löst, unvollständig wäre. Es ist jedoch völlig unentscheidbar, ob uns die Aussagen, die wahr sind, aber nicht bewiesen werden können, stören sollten, da Gödels Beweis ein sehr dummes Beispiel für Inkonsistenz konstruiert (ungefähr vom Typ "Ich bin falsch").
Die Gödel-Nummerierung im formalen Beweis ist nicht besonders albern oder trivial, nur weil sie rekursiv ist, auch sind die Aussagen nicht unbedingt wahr oder falsch und könnten möglicherweise ein unbekannter oder unverständlicher Wert oder eine Eigenschaft sein.
Sie scheinen auch das gegebene Problem zu trivialisieren, zum Beispiel kann man angesichts des der Quantenmechanik äquivalenten Axiomsystems beweisen, dass diese Theoreme der Gravitationstheorien (allgemeine / spezielle Relativitätstheorie) innerhalb des Systems möglicherweise nicht bewiesen oder widerlegt werden, und das umgekehrte Ergebnis für das Axiom System, das den Gravitationstheorien (allgemeine/spezielle Relativitätstheorie) entspricht, und beweist damit, dass es keine Auflösung dieser beiden Theorien im selben Rahmen gibt und wie in der Eingangsfrage angegeben, möglicherweise keine universelle Theorie der Physik ("Theorie von allem") existiert.
@ACuriousMind Dies ist kein Beispiel für eine Inkonsistenz, sondern für eine unentscheidbare Aussage. Ein viel weniger dummes Beispiel für eine unentscheidbare Aussage ist die Konsistenz (eines ausreichend starken formalen Systems) selbst. Kein tatsächliches Beispiel für Inkonsistenz wäre dumm, da sich daraus jede Aussage als Theorem ableiten lässt.
@doetoe: Ah, tut mir leid, es hat eine Weile gedauert, bis ich mich mit formaler Logik befasst habe. Ich meinte "dumm" in dem Sinne, dass es nicht unmittelbar ist, dass jede unentscheidbare Aussage tatsächlich eine Aussage über die physische Welt wäre - es sind höchstwahrscheinlich "Meta-Aussagen" (wie die Konsistenz des Systems selbst, wie Sie sagen, auch Ist). An OP: Was Sie beschreiben, könnte passieren, da wir keine Ahnung haben, wie ein formales Axiomensystem für die Physik tatsächlich aussehen würde. Wenn die Frage lautet: "Könnte es sein, dass es keine Theorie von allem gibt?" Die Antwort ist (zumindest für mich offensichtlich) "Ja"..
Ist es jedoch möglich, die Negation der Existenz der Theorie von allem zu beweisen?
Die Eröffnungsaussage, dass C eine Teilmenge von U ist, erscheint höchst fragwürdig, insbesondere ohne genaue Definitionen dessen, was die Physik mit "Konzept" und "Codierung" meinen würde. Ich denke, dieses Problem macht die gesamte Argumentationslinie unpräzise und relativ nutzlos. Zum Beispiel ist das Konzept des "Rauschens" wirklich sehr wichtig für Berechnungen, wird aber von allen mathematischen Berechnungsmodellen ignoriert. Und auch von Logic ignoriert. Und von Gödel.

Antworten (2)

Die Existenz unentscheidbarer Aussagen bedeutet nicht, dass es uns nicht gelingen könnte, hypothetisch unter Hinzufügung geeigneter widerspruchsfreier Axiome die Wahrheit aller physikalisch relevanten Aussagen zu bestimmen. Angenommen, Sie haben eine Theorie L mit einer sinnvollen unentscheidbaren Aussage A , so dass beides L + A Und L + A ¯ (ich benutze bar für Negation) sind konsistent. Dann kannst du beides wählen L + A oder L + A ¯ als Ihre neue Theorie (je nachdem, was physikalisch relevanter ist).

Die unentscheidbare Aussage von Gödel zum Beispiel scheint (meiner Meinung nach) für eine physikalische Theorie nicht sehr relevant zu sein, und auch die Tatsache, dass die Konsistenz der Theorie darin nicht beweisbar ist (da Sie im Prinzip Erzwingen / Imprädikativität zum Beweis verwenden können Konsistenz zu finden und Wahrheitskomplexitäten des logischen Systems zu finden, auch wenn es bedeuten würde, mathematische Entitäten zu berücksichtigen, die nicht „physisch“ sind).

Als konkretes Beispiel: Würde es die Physik interessieren, ob die verallgemeinerte Kontinuumshypothese wahr oder falsch ist? G C H ist unentscheidbar in Z F C , Und Z F C , Z F C + G C H , Z F C + G C H ¯ sind konsistente Theorien. Ich nehme an, es ist nicht so wichtig, welches man für die Physik wählt, aber Sie brauchen sicherlich die Mengenlehre. Das Axiom der Wahl ist stattdessen relevanter, da es ein Modell von gibt Z F + D C (zuverlässige Wahl), bei der alle Sätze von Realzahlen messbar sind (aber andererseits fällt es Ihnen schwer, Verteilungen zu definieren), und das kann physikalisch relevanter sein (vielleicht ... ich weiß es nicht).

Der Unvollständigkeitssatz gilt immer noch für (L + A L + ¬ A ) es spielt also keine Rolle, wie viele unentscheidbare Aussagen Sie als Axiome hinzufügen (vorausgesetzt, Sie kennen irgendwie ihren Wahrheitswert). Was die "willkürliche" Beziehung zwischen (der Relevanz) eines Theorems für die Physik im Allgemeinen betrifft (um nicht zu sehr auf die Philosophie einzugehen), wenn es nicht relevant ist, hält sich das Axiomensystem offensichtlich nicht an die darauf definierten Einschränkungen (irgendwie physikalische Realität und ihre Modelle transzendieren die Logik), dann ist es kein Axiomensystem mehr, also keine "Axiomatisierung der Physik" mehr
Auch da jede beliebige Gleichung in ein Axiomensystem umgewandelt (darauf reduziert) werden kann, wenn wir davon ausgehen, dass eine Theorie von allem existiert und sie in Form einer beliebigen einzelnen Gleichung vorliegt, nehmen wir sie als Axiom (oder mehrere, es spielt keine Rolle). ) Da die Realität nicht an die Gesetze gebunden ist, die diesem Axiom (oder einem Axiomensystem) entsprechen, ist sie daher keine Theorie von allem und nicht einmal ein Modell der Realität.
Erstens ist es nicht wahr, dass sich eine Gleichung auf ein Axiomensystem reduziert; Eine physikalische Theorie (oder ein Realitätsmodell) wird innerhalb eines gegebenen logischen Modells formuliert, wenn Sie sonst keine Schlussfolgerungen ziehen und daher Ihr Wissen verbessern können. was ich sagen will, ist, dass Sie, auch wenn es immer unentscheidbare Aussagen geben wird, Ihrem System im Prinzip die Wahrheit der physikalisch relevanten aufzwingen und die Konsistenz der Theorie mit den logischen Werkzeugen als Forcen überprüfen können. Sie werden also meiner Meinung nach nicht durch Unentscheidbarkeit daran gehindert, eine vollständige Axiomatisierung der Physik zu erhalten
und eine Theorie von allem wäre sowieso ein Modell, dh ein logisch/mathematisches System (mit einer riesigen unabzählbaren Wahrheitskomplexität), so dass jede physikalisch relevante Aussage beweisbar ist. davon sind wir natürlich weit entfernt...
Wenn eine physikalische Theorie innerhalb eines logischen Modells formuliert wird, dann wären jene Axiome und Folgerungsregeln, aus denen die Gleichungen und physikalischen Modelle abgeleitet würden, eine Axiomatisierung der Physik. Darüber hinaus muss (unter der Annahme, dass es eine einzige vereinheitlichende Gleichung gibt) angenommen werden, dass eine einzelne Gleichung "wahr" (ein Axiom) ist, und verschiedene algebraische oder mathematische Manipulationen wären Theoreme oder Lemmata und diese mathematischen Regeln, die verwendet werden, um die Gleichung zu manipulieren und diese Theoreme abzuleiten ( Vorkommen von Variablen bestimmte Ableitungen, Ungleichungen etc.)
wären die Schlußregeln, die das Axiomensystem begleiten, und somit ist die Gleichung (das Axiomensystem, zu dem sie äquivalent ist) immer noch an den Unvollständigkeitssatz gebunden, und man würde irgendeine Form von beweistheoretischer Ordinalanalyse oder ein neues Feld benötigen, um ihre Konsistenz zu beweisen und Vollständigkeit jedoch wäre das System theoretisch in der Lage, jede wahre Ableitung oder jedes Ereignis davon zu beweisen, das in der physikalischen Welt auftreten könnte, und wäre daher diese beweistheoretische Ordinalanalyse nicht innerhalb des Systems (Theorie aller Physik) und an Gödels Theorem gebunden?
Ich fürchte, Ihnen ist nicht so klar, was ein logisches System ist. Es ist eine Sprache und ihre Regeln (wie Englisch); und Sie könnten keine Gleichung schreiben, ohne die Sprache zu definieren. Es ist viel grundlegender, es ist der Apparat, in dem Sie Ihr physikalisches Modell der Realität formulieren könnten (denn darum geht es in der theoretischen Physik). Und die Aussagen der logischen Theorie sind in der Theorie zulässige Sätze und somit wäre ihr Ensemble weit größer als die Sätze über Physik. Und unter diesen Sätzen werden einige unentscheidbar sein.
Wenn wir die unentscheidbaren Sätze nur nicht-physikalisch sein lassen könnten, könnten wir im Prinzip zufrieden sein. Offensichtlich können wir das jetzt nicht.
Sie missverstehen, die Regeln, die die Zeichenfolgen der Sprache einschränken, wären die Gleichung (en) (daher würden die Gleichung (en) die Sprache definieren). Daher wären alle ableitbaren Zeichenketten/Aussagen physikalische Aussagen.
Ich bin nicht davon überzeugt, dass die Physik die Mengenlehre braucht. Nur messbare Mengen sind physisch. Ich bin mir auch nicht sicher, ob die Physik die gesamte Logik höherer Ordnung benötigt. Vielleicht braucht es nur Logik erster Ordnung?
@josephf.johnson Die übliche Mengenlehre ist eine Theorie der Logik erster Ordnung. Und was meinen Sie mit messbaren Mengen? Wie auch immer, Mathematik basiert auf Mengenlehre und Logik. Wer also die physikalische Welt mathematisch beschreiben will, braucht Mengenlehre und Logik. Sie dürfen sie nicht explizit verwenden, aber sie sind immer impliziert ...

Ich finde es interessant, diese Frage vor dem Hintergrund eines sehr einfachen hypothetischen Universums zu stellen. Angenommen, das Universum bestünde beispielsweise ausschließlich aus vollkommen harten Kugeln, die sich nach klassischer Physik bewegen und elastisch aufeinanderprallen. (Wirklich, die spezifischen Regeln unseres Spielzeuguniversums spielen für das, was ich sagen werde, keine Rolle, solange sie einfach und eindeutig sind).

Soweit ich das beurteilen kann, hindert uns nichts daran, dieses Spielzeuguniversum so rigoros zu modellieren, wie wir es wünschen. Aber wäre unser Modell eine konsistente, vollständige Theorie? Und wenn ja, was ist mit Gödels Theorem?

Der Satz von Gödel gilt für formale Systeme, in denen Zeichenketten Aussagen, Beweise usw. darstellen können. Das Spielzeuguniversum ist selbstverständlich kein formales System. Auch die Kenntnis der mathematischen Regeln des Spielzeuguniversums sagt uns nicht sofort, was das formale System ist. Was ist also unser formales System, und was noch wichtiger ist, was wird die Semantik sein, mit der wir seine Fäden auf Fakten über das Spielzeuguniversum abbilden? Und unter welchen Bedingungen wird das System als "vollständige" Beschreibung des Spielzeuguniversums betrachtet? Wir müssen uns auf genaue Antworten auf diese Fragen einigen. Ansonsten glaube ich, dass die Antwort auf die Hauptfrage nur lauten kann: "es kommt darauf an, was Sie meinen."

Wenn wir in diesem Spielzeuguniversum leben würden, wären wir dann berechtigt zu sagen, dass wir eine „Theorie von allem“ hätten?

Es ist richtig, dass ein Teil der in der Theorie verwendeten Mathematik Arithmetik wäre, und daher gäbe es unentscheidbare Aussagen über Arithmetik. Keine Überraschung. Wie Yuggib sagt, sind diese physikalisch nicht relevant. Es ist uns egal, dass unsere „Spielzeuguniversum-Theorie von allem“ uns nicht sagen kann, ob eine diophantische Gleichung Lösungen hat oder nicht, wenn sie sich nicht auf die Physik des Spielzeuguniversums bezieht. Was wäre, wenn es so wäre? Was wäre, wenn wir aus harten Sphären einen schrecklich komplizierten "Computer" zusammenbrauen würden, dessen Verhalten auf die diophantische Gleichung abgebildet und nicht vorhergesagt werden könnte? Wäre das eine „Unvollständigkeit“ des Modells?

Nun, das überlasse ich Ihnen zu entscheiden. Ich persönlich betrachte ein mathematisches Modell eines physikalischen Systems als "vollständig", wenn es uns erlaubt, Berechnungen im Prinzip mit jeder gewünschten Genauigkeit durchzuführen. Wenn wir also in einem Universum der harten Sphäre lebten, würde ich sagen, dass wir eine „Theorie von allem“ besäßen. Und außerdem, dass jedes a priori Argument, das eine „Theorie von allem“ für das reale Universum ausschließt, erklären sollte, in welcher wesentlichen Weise es sich von dem Spielzeug unterscheidet.

Natürlich steht es einem frei, eine andere Definition von „Vollständigkeit“ zu wählen und zu sagen, dass ein Modell nur dann vollständig ist, wenn die detaillierten Ergebnisse bestimmter Konfigurationen/Systeme/Situationen beweisbar sind. In diesem Fall wird es niemals eine "Theorie von allem" für ein so einfaches Universum wie eine Turing-Maschine geben!

Ich meine vollständig im Sinne der mathematischen Logik, dh die maximal konsistente Menge von Sätzen, die die physikalische Realität modellieren. Zum Beispiel würde man in Ihrem Spielzeuguniversum die Grundgleichungen der klassischen Physik als Axiome und Manipulationen gemäß den Schlußregeln nehmen (nehmen Sie die Schlußregeln als die der Algebra (Assoziativität usw.), die der mathematischen Analyse und aller anderen relevanten mathematischen Konstrukte). als Theoreme und Lemmata usw. Da die Theorie vollständig ist, wären diese Manipulationen repräsentativ für physikalische Phänomene und würden im Universum zutreffen.
Ich nehme an, die Frage ist dann, ob es sich lohnt, die Negation der Existenz einer "Theorie von allem" mit diesen Fakten in einem Papier zu formalisieren, oder wird es nur eine nutzlose Tautologie sein.
Nun, der Computer würde dann wahrscheinlich einfach ewig brauchen, um Lösungen für die Gleichung zu berechnen, und die Aussage von Gödels Theorem, auf diesen Computer angewendet, würde einfach zu einer Aussage des "Ist es fertig?" Problem.
@ Jerry Schirmer Die unentscheidbaren Aussagen, die sich aus dem Halteproblem ergeben, gelten für den Algorithmus, der den Implikationsregeln und dem Axiomensystem für U={x|∃x} entspricht, mit einer Eingabe, die einem Theorem entspricht, um unter Verwendung dieser auf diese angewendeten Implikationsregeln zu beweisen Axiome (das ist der Algorithmus) ist äquivalent zu den unentscheidbaren Aussagen von Gödels Unvollständigkeitssatz beweist, dass sie unentscheidbar sind (Konsistenz vorausgesetzt), und die Frage beweist auch, dass sie zu demselben Paradoxon führen (allerdings trivial unabhängig von der Reduzierbarkeit des einen auf das andere (Äquivalenz). kann auch beweisen, wenn ...
... ( A ich ) ist gleichbedeutend mit der Aussage, dass der Algorithmus A eine gegebene Eingabe i anhält (trivialerweise bedeutet dies, dass das Halteproblem für eine gegebene Eingabe i entscheidbar ist) und A der Algorithmus ist, der aus dem Axiomensystem für U mit den Implikationsregeln konstruiert ist, und i ein Theorem ist es ist dann offensichtlich mit Hilfe der Implikationsregeln zu beweisen ( ich ( ¬ A ich ) ) ( ich ( ich U ) ) ( ich ( ¬ ich ) ) das ist das gleiche Paradoxon wie in der Frage